0

Sum to infinite terms of a gp - class-XI

Description: sum to infinite terms of a gp
Number of Questions: 68
Created by:
Tags: maths sequence, progression and series
Attempted 0/67 Correct 0 Score 0

The value of $3 - 1 + \frac{1}{3} - \frac{1}{9} +  \ldots $ is equal to

  1. $\dfrac{{20}}{9}$

  2. $\dfrac{{9}}{20}$

  3. $\dfrac{{9}}{4}$

  4. $\dfrac{{4}}{9}$


Correct Option: C

Let $P = 3^{1/3} . 3^{2/9} . 3^{3/27} ...\infty$, then $P^{1/3}$ is equal to

  1. $3^{2/3}$

  2. $\sqrt {3}$

  3. $3^{1/3}$

  4. $3^{1/4}$


Correct Option: D
Explanation:

Above in an infinite A.G.S. with $a = 1, d = 1$ for A.P., $b = \dfrac {1}{3}, r = \dfrac {1}{3}$ for G.P.
$\therefore S _{\infty} = \dfrac {ab}{1 - r} + \dfrac {dbr}{(1 - r)^{2}} = \dfrac {\dfrac {1}{3}}{1 - \dfrac {1}{3}} + \dfrac {1 . \dfrac {1}{3} . \dfrac {1}{3}}{\left (1 - \dfrac {1}{3}\right )^{2}} = \dfrac {1}{2} + \dfrac {1}{4} = \dfrac {3}{4}$
$\therefore P = 3^{S} = 3^{3/4} \therefore P^{1/3} = 3^{1/4}$.

The first term of a $G.P.$ whose second term is $2$ and sum to infinity is $8$ will be

  1. $6$

  2. $3$

  3. $4$

  4. $1$


Correct Option: C
Explanation:

$ar = 2, \dfrac {a}{1 - r} = \dfrac {1}{8}$. Eliminate $r$ and we get
$a^{2} - 8a + 16 = 0 \therefore (a - 4)^{2} = 0$ or $a = 4$

The value of $9^{1/3}\times 9^{1/9} \times 9^{1/27} \times .....\infty$ is

  1. $9$

  2. $1$

  3. $3$

  4. None of these


Correct Option: C
Explanation:

$(9)^{S _{\infty}} = 9^{1/2} = 3\because S _{\infty} = \dfrac {1/3}{1 - (1/3)} = \dfrac {1}{2}$.

The value of $9^\cfrac{1}{3}.9^\cfrac{1}{9}.9^\cfrac{1}{27}...........$ upto $\infty$, is

  1. $1$

  2. $3$

  3. $9$

  4. $None\ of\ these$


Correct Option: B
Explanation:
${9^{\cfrac{1}{3}}}{.9^{\cfrac{1}{9}}}{.9^{\cfrac{1}{{27}}}} -  -  -  - upto\,\,\infty $
$ = {9^{\left( {\cfrac{1}{3} + \cfrac{1}{9} + \cfrac{1}{{27}} +  -  -  - } \right)}}$
$ = {9^{\left( {\cfrac{{\cfrac{1}{3}}}{{1 - \cfrac{1}{3}}}} \right)}}$
$ = {9^{\left( {\cfrac{{\cfrac{1}{3}}}{{\cfrac{2}{3}}}} \right)}}$
$ = {9^{\cfrac{1}{2}}}$
$ = 3$

If $x=1+a+{ a }^{ 2 }+{ a }^{ 3 }+....$ to $\infty \left( \left| a \right| <1 \right) $ and 
$y=1+b+{ b }^{ 2 }+{ b }^{ 3 }+...$ to $\infty \left( \left| b \right| <1 \right) $ then
$1+ab+{ a }^{ 2 }{ b }^{ 2 }+{ a }^{ 3 }{ b }^{ 3 }+...$ to $\infty =\cfrac { xy }{ x+y-1 } $

  1. True

  2. False


Correct Option: A
Explanation:

${ x }=\dfrac { 1 }{ 1-a } $ ${ y }=\dfrac { 1 }{ 1-b } $ [summing infinite $G.P's$].
$\therefore a=\dfrac { x-1 }{ x } $, $b=\dfrac { y-1 }{ y } $
$\therefore 1+ab+{ a }^{ 2 }{ b }^{ 2 }+...\infty $
$=\dfrac { 1 }{ 1-ab } =\dfrac { 1 }{ 1-\dfrac { (x-1)(y-1) }{ xy }  } =\dfrac { xy }{ x+y-1 }. $

The sum to infinity of the series $1 + \dfrac{2}{3} + \dfrac{6}{{{3^2}}} + \dfrac{{10}}{{{3^3}}} + \dfrac{{14}}{{{3^4}}} + ......,is$

  1. $3$

  2. $4$

  3. $6$

  4. $2$


Correct Option: A
Explanation:
Let $S$=$1 + \cfrac{2}{3} + \cfrac{6}{{{3^2}}} + \cfrac{{10}}{{{3^3}}} + ......$

$\cfrac{S}{3} = \cfrac{1}{3} + \cfrac{2}{{{3^2}}} + \cfrac{6}{{{3^3}}} + ....$

$S - \cfrac{S}{3} = 1 + \cfrac{1}{3} + \cfrac{4}{{{3^2}}} + ....$

$\cfrac{{2S}}{3} = \cfrac{{\cfrac{4}{3} }}{{1 - \cfrac{1}{3}}}$

$ = \dfrac{\cfrac{4}{3}} { \cfrac{2}{3}} = \cfrac{4}{2} = 2$

If $x = 1\, + a + {a^2} + ......\infty $, $y = 1\, + b + {b^2}\,\, + ......\infty $ where $\left| a \right| < 1$ and $\left| b \right| < 1$, then $\left( {1 + ab + {a^2}{b^2} + ........\infty } \right) = ?$

  1. $\frac{xy}{x+y}$

  2. $\frac{x+y}{xy}$

  3. $\frac{xy}{x+y+1}$

  4. $\frac{xy}{x+y-1}$


Correct Option: D

Value of $y = {\left( {0.64} \right)^{{{\log } _{0.25}}\left( {\cfrac{1}{3} + \cfrac{1}{{{3^2}}} + \cfrac{1}{{{3^3}}}....upto   \infty } \right)}}$ is :

  1. $0.9$

  2. $0.8$

  3. $0.6$

  4. $0.25$


Correct Option: B
Explanation:
$y= (0.64)^{log _{0.25} \left(\dfrac{1}{3}+ \dfrac{1}{3^{2}}+ \dfrac{1}{3^{3}}+..... \right)}$
$=(0.64)^{\log _{0.25}^{\left( \dfrac{\dfrac{1}{3}}{1-1/3} \right)}}$
$=(0.64)^{\log _{0.25} } \left( \dfrac{1}{2} \right)$
$= (0.64)^{\log 0.5} _{0.25}$
$(0.64)^{0.5}= (0.64)^{1/2}= \sqrt{0.64}= 0.8$

If $y=x-x^2+x^3-x^4+....\infty$, then value of x will be?

  1. $y+\dfrac{1}{y}$

  2. $\dfrac{y}{1+y}$

  3. $y-\dfrac{1}{y}$

  4. $\dfrac{y}{1-y}$


Correct Option: D

If the sum of the series $2+\frac {\displaystyle 5}{\displaystyle x}+\frac {\displaystyle 25}{\displaystyle x^2}+\frac {\displaystyle 125}{\displaystyle x^3}+....$ is finite, then-

  1. $\mid x\mid > 5$

  2. -5 < x < 5

  3. $\mid x\mid < 5/2$

  4. $\mid x\mid > 5/2$


Correct Option: A
Explanation:

We can rewrite the series as
$1+1+\dfrac {5}{x}+(\dfrac {5}{x})^2+(\dfrac {5}{x})^3+.....$
We can sum up this series if $\mid 5/x\mid < 1$
$\Leftrightarrow \mid x\mid > 5$

If $x=1+a+a^2+...\infty$ where $|a| <1 $ and $y=1+b+b^2+...\infty$, where $|b| < 1$, then $1+ab+a^2b^2+...\infty =\dfrac{xy}{x+y-1}$.

  1. True

  2. False


Correct Option: A

${x}^{\cfrac{1}{2}}.{x}^{\cfrac{1}{4}}.{x}^{\cfrac{1}{8}}.{x}^{\cfrac{1}{16}}.....$ to $\infty$

  1. $0$

  2. $1$

  3. $x$

  4. $\infty$


Correct Option: C
Explanation:

$x^{1/2}.x^{1/4}......\infty =x^{\left ( \frac{1}{2}+\frac{1}{4}+\frac{1}{8}+\frac{1}{16} \right )}$ 


$=x\left ( 1/2+1/2^{2}+1/2^{3}+1/2^{4}+.......\infty  \right )$

$=x^{\frac{1}{2}\left ( \frac{1-(1/2)\infty }{1-1/2} \right )}$ 

$=x^{1}$ 

$=x$

The solution of the equation $(8)^{1+|cos x|+|cos x|^2+|cos x|^3+...)}=4^3$ in the interval $(-\pi, \pi)$ are.

  1. $\pm \dfrac {\pi }{3}, \pm \dfrac {\pi }{6}$

  2. $\pm \dfrac {\pi }{3}, \pm {\pi }$

  3. $\pm \dfrac {\pi }{3}, \pm \dfrac {2\pi }{3}$

  4. none of these


Correct Option: C
Explanation:

 $(8)^{1+|cos x|+|cos x|^2+|cos x|^3+...)}=4^3$
$\Rightarrow (8)^{\dfrac{1}{1-|\cos x|}}=4^3=64=8^2$, since $|\cos x| < 1$ in $(-\pi, \pi)$
$\Rightarrow {\dfrac{1}{1-|\cos x|}}=2$
$\Rightarrow |\cos x|=\cfrac{1}{2}$
The solution in the given interval is,
$x=\pm \cfrac{\pi}{3}, \pm \cfrac{2\pi}{3}$
Hence, option 'C' is correct.

The sum of $7+1+.......$

  1. $\dfrac{49}{6}$

  2. $\dfrac{49}{8}$

  3. $\dfrac{49}{14}$

  4. None of these


Correct Option: A
Explanation:

$7+1+\dfrac 17 ...$

Forms GP  with $a=7\r=\dfrac 17 $
So $S _{\infty}=\dfrac{7}{1-\dfrac 17}\\dfrac{49}{6}$

The series $\dfrac{2x}{x+3}+(\dfrac{2x}{x+3})^{2}+(\dfrac{2x}{x+3})^{3}+........\infty$ will have a definite sum when  

  1. $x<3$

  2. $x>3$

  3. $x=0$

  4. $x=-3$


Correct Option: A
Explanation:
$\dfrac{2x}{x+3}+\left(\dfrac{2x}{x+3}\right)^{2}+\left(\dfrac{2x}{x+3}\right)^{3}.......\infty $
$\therefore a=\dfrac{2x}{x+3}$   $r=\dfrac{2x}{x+3}$
$\therefore s=\dfrac{a}{1-r}=\dfrac{\dfrac{2x}{x+3}}{1-\dfrac{2x}{x+3}}$
$=\dfrac{2x}{x+3-2x}=\dfrac{2x}{3-x}$
Now, to have definite sum
$r < 1$
$\therefore \dfrac{2x}{x+3} < 1$
$\therefore 2x < x+3$
$\therefore x < 3$

Find the sum of $4,2,1,\cdots$ 

  1. 8

  2. 16

  3. 32

  4. 64


Correct Option: A
Explanation:

The series $4,2,1,\cdots$ form GP

With $a=4$ and $r=\dfrac 12$
$S _{\infty}=\dfrac{a}{1-r}\\dfrac{4}{1-\dfrac 12 }=\dfrac{4}{\dfrac 12}=4\times 2=8$

If $y=x^{\dfrac {1}{3}}.x^{\dfrac {1}{9}}.x^{\dfrac {1}{27}}......\infty $, then $y =$

  1. $x^{1/3}$

  2. $x^{2/3}$

  3. $x^{1/2}$

  4. $x$


Correct Option: C
Explanation:

$y=x^{\dfrac {1}{3}}.x^{\dfrac {1}{9}}.x^{\dfrac {1}{27}}......\infty $
 $ =x^{\cfrac{1}{3}+\cfrac{1}{3^2}+\cfrac{1}{3^3}+........\infty }=x^{\cfrac{1/3}{1-1/3}}=x^{1/2}$
Hence, option 'C' is correct.

If sum of an infinite geometric series is $\dfrac{4}{3}$ and its Ist term is $\dfrac{3}{4}$, then its common ratio is

  1. $\dfrac{7}{16}$

  2. $\dfrac{9}{16}$

  3. $\dfrac{1}{9}$

  4. $\dfrac{7}{9}$


Correct Option: A
Explanation:

We know that sum of infinite geometric series $=\dfrac{a}{1-r}$
Where $a=\text{first term}$ and $r=\text{common ratio}$.
$\dfrac{a}{1-r}=\dfrac{4}{3}$
Then, $\dfrac{\dfrac{3}{4}}{1-r}=\dfrac{4}{3}\Rightarrow\,r=1-\dfrac{9}{16}=\dfrac{7}{16}$

The value of x that satisfies the relation 
$x=1-x+{ x }^{ 2 }-{ x }^{ 3 }+{ x }^{ 4 }-{ x }^{ 5 }+........\infty $ 

  1. $2cos{ 3 }6^{ \circ }$

  2. $2cos144^{ \circ }$

  3. $2sin18^{ \circ }$

  4. none


Correct Option: A
Explanation:

The series $1-x+x^2-....$ form $GP$ with $a=1 ,r=-x$

Sum of infinte GP is $x=\dfrac{a}{1-r}\x=\dfrac{1}{1+x}\x+x^2=1\x^2+x-1=0$
By quadratic formulae 
$x=\dfrac{-1\pm\sqrt{1+4}}2\2\dfrac{-1\pm\sqrt5}{4}\2\cos 36^{\circ}$

If $x>0$ and $\displaystyle log _{2}x+log _{2}(\sqrt{x})+log _{2} (\sqrt[4]{x})+log _{2}(\sqrt[8]{x})+...\infty =4 ,$then $x=$

  1. 2

  2. 3

  3. 4

  4. 5


Correct Option: C
Explanation:

Given $log _{2}x+log _{2}(\sqrt{x})+log _{2} (\sqrt[4]{x})+log _{2}(\sqrt[8]{x})+...\infty =4 $

$\Rightarrow log _{2}[x.x^{1/2}.x^{1/4}.x^{1/8}...\infty

]=log _{2}[x^{1+\dfrac{1}{2}+\dfrac{1}{4}+\dfrac{1}{8}+...\infty} ]=log _{2}

x^{\dfrac{1}{1-(1/2)}}=log _{2}(x^{2})=4 $

$ \therefore x^{2}=2^{4}=16

 \therefore x=4$

What is the sum of the series $ 1 - \frac{1}{2} + \frac{1}{4} - \frac{1}{8} + ....$ equal to ?

  1. $\dfrac{1}{2}$

  2. $\dfrac{3}{2}$

  3. $2$

  4. $\dfrac{2}{3}$


Correct Option: D
Explanation:
$1,\dfrac { -1 }{ 2 } ,\dfrac { 1 }{ 4 } ,\dfrac { -1 }{ 8 } ,..$ is in G.P series. 
So, sum of infinite terms of a G.P is $\dfrac { a }{ 1-r } $, where $a$ is first term $=1$
$r$ is common difference $=-1/2=-0.5$
Thus, $1+\dfrac { -1 }{ 2 } +\dfrac { 1 }{ 4 } +\dfrac { -1 }{ 8 } ,..=$ $\dfrac { 1 }{ 1-\left( \frac { -1 }{ 2 }  \right)  } =\dfrac { 1 }{ \left( \frac { 3 }{ 2 }  \right)  } =\dfrac { 2 }{ 3 } $
Hence, D is correct.

The sum of the series formed by the sequence $3, \sqrt{3}, 1....... $ upto infinity is : 

  1. $\frac {3\sqrt{3}(\sqrt{3}+1)}{2}$

  2. $\frac {3\sqrt{3}(\sqrt{3} - 1)}{2}$

  3. $\frac {3(\sqrt{3}+1)}{2}$

  4. $\frac {3(\sqrt{3}-1)}{2}$


Correct Option: A
Explanation:

For the given series

$\ \cfrac { \sqrt { 3 }  }{ 3 } =\cfrac { 1 }{ \sqrt { 3 }  } \$
So it is a GP with common ratio $\cfrac{1}{\sqrt {3}}$
Formula for sum of infinite terms of GP $ =\cfrac { a }{ 1-r }$,  where
$a$ is first term and r is the common ratio
$\Rightarrow  { S } _{ \infty  }=\cfrac { 3 }{ 1-\cfrac { 1 }{ \sqrt { 3 }  }  }  =\cfrac { 3\sqrt { 3 }  }{ \sqrt { 3 } -1 }$

$=\cfrac { 3\sqrt { 3 } (\sqrt { 3 } +1) }{ (\sqrt { 3 } -1)(\sqrt { 3 } +1) }$ ..... [On rationalizing]

$ =\cfrac { 3\sqrt { 3 } (\sqrt { 3 } +1) }{ 2 } $
Hence, A is correct.

In a Geometric progression with common ratio less than $1$, if $n$ approaches $\infty$ then ${ S } _{ \infty  }$ is

  1. $a{ r }^{ 0 }$

  2. $a{ r }^{ n-1 }$

  3. $\cfrac { 1-r }{ a } $

  4. $\cfrac { a }{ 1-r } $


Correct Option: D
Explanation:

$S _{n}=\dfrac{a(1-r^n)}{(1-r)}$

Now, as $n$ tends to $\infty$ then $r^n$ tends to $0$
$\therefore$ $S _{\infty}=\dfrac{a(1-r^{\infty})}{(1-r)}$
           $=\dfrac{a}{1-r}$       $(\because\displaystyle \lim _{n\rightarrow \infty}r^{\infty} = 0$ for $r<1)$
Hence, $S _{\infty}=\dfrac{a}{(1-r)}$

Find the sum of the infinite geometric series $1+\dfrac{1}{2}+\dfrac{1}{4}+\dfrac{1}{8}+.......$

  1. $16$

  2. $14$

  3. $-11$

  4. $2$


Correct Option: D
Explanation:
Given sequence is $1+\dfrac {1}{2}+\dfrac {1}{4}+\dfrac {1}{8}+....$
Thus $a=1$, $r = \dfrac{1}{2}$
Therefore, $\text{sum} =\dfrac{a}{1-r}$
$\Rightarrow \text{sum} = \dfrac{1}{1-\frac{1}{2}}$
$\Rightarrow \text{sum} =2$

If $p$ is positive, then the sum to infinity of the series, ${1 \over {1 + p}} - {{1 - p} \over {{{(1 + p)}^2}}} + {{{{(1 - p)}^2}} \over {{{(1 + p)}^3}}} - ......$ is

  1. $1/2$

  2. $3/4$

  3. $1$

  4. None of these


Correct Option: A
Explanation:
$a=\dfrac { 1 }{ 1+P } \\ r=-\dfrac { 1-P }{ 1+P } $
Sum to infinity $=\dfrac { a }{ 1-r } \\ =\dfrac { \dfrac { 1 }{ 1+P }  }{ 1+\dfrac { 1-P }{ 1+P }  } \\ =\dfrac { \dfrac { 1 }{ 1+P }  }{ \dfrac { 1+P+1-P }{ 1+P }  } =\dfrac { 1 }{ 2 } $

If $f(x) = x - {x^2} + {x^3} - {x^4} + .............\infty $ where $\left| x \right|\langle 1$ then ${f^{ - 1}}(x) = $

  1. ${\dfrac{x}{1 - x}}$

  2. ${\dfrac{x}{1 + x}}$

  3. ${\dfrac{1}{1 - x}}$

  4. ${\dfrac{1}{1 + x}}$


Correct Option: A
Explanation:

$f\left( x \right) =\dfrac { x }{ 1+x } \ x=\dfrac { f^{ -1 }\left( x \right)  }{ 1-f^{ 1 }\left( x \right)  } \ \therefore f^{ 1 }\left( x \right) =\dfrac { x }{ 1-x } $

If the sum of an infinitely decreasing G.P. is $3$, and the sum of the squares of its terms is $\dfrac {9}{2}$, then the sum of the cubes of the terms is

  1. $\dfrac {105}{13}$

  2. $\dfrac {108}{13}$

  3. $\dfrac {729}{8}$

  4. $\dfrac {108}{9}$


Correct Option: B
Explanation:
Let the GP be $a,ar,ar^2,ar^3,...$
The first term be $a$ and the common ratio be $r$. 
Then, it is given that:
$\text{Sum}=\dfrac{a}{1-r} = 3$    ....(1)
Sequence of squares of terms is $a^2, a^2r^2, a^2r^4,...$
$\text{Sum}=\dfrac{a^2}{1-r^2} = \dfrac{9}{2}$    ....(2)
Thus, we have:
$\dfrac{a^2\div a}{(1-r^2)\div (1-r)} = \dfrac{9 \div 3}{2} $
$\Rightarrow \dfrac{a}{1+r} = \dfrac{3}{2}$    ....(3)

Now divide equations (1) and (3),
$\dfrac{1-r}{1+r} = \dfrac{1}{2}$
$ \Rightarrow r = \dfrac{1}{3}$
Substituting this value of $r$ in equation (1), we get $a = 2$
Therefore, $\dfrac{a^3}{1-r^3} = \dfrac{8}{\left (1-\dfrac{1}{27}\right)} = \dfrac{108}{13}$
Thus, the answer is option B.

Sum of the series ${9^{{1 \over 3}}} \times {9^{{1 \over 9}}} \times {9^{{1 \over {27}}}} \times .......$  is equal to

  1. $3$

  2. $9$

  3. $27$

  4. $81$


Correct Option: A
Explanation:

${ 9 }^{ \cfrac { 1 }{ 3 }  }\times { 9 }^{ \cfrac { 1 }{ 9 }  }\times { 9 }^{ \cfrac { 1 }{ 27 }  }\times ...\infty $

$={ 9 }^{ \cfrac { 1 }{ 3 }  +\cfrac { 1 }{ 9 } +\cfrac { 1 }{ 27 } + ...}$
Let $S=\cfrac { 1 }{ 3 } +\cfrac { 1 }{ 9 } +\cfrac { 1 }{ 27 } +...$
$=\cfrac { \cfrac { 1 }{ 3 }  }{ 1-\cfrac { 1 }{ 3 }  } $
$=\cfrac { \cfrac { 1 }{ 3 }  }{ \cfrac { 2 }{ 3 }  } =\cfrac { 1 }{ 2 } $
$\therefore { 9 }^{ \cfrac { 1 }{ 3 }  }\times { 9 }^{ \cfrac { 1 }{ 9 }  }\times { 9 }^{ \cfrac { 1 }{ 27 }  }\times ...\infty ={ 9 }^{ \cfrac { 1 }{ 2 }  }$
$=(3^{ 2 })^{ \cfrac { 1 }{ 2 }  }$
$=3$

If the expansion in powers of x of the function $\dfrac{1}{(1 - ax)(1 - bx)} , (a \neq b)$ is $a _0 + a _1x + a _2x^2 + .... \, then \, a _n$ is

  1. $\dfrac{b^n - a^n}{b - a}$

  2. $\dfrac{a^n - b^n}{b - a}$

  3. $\dfrac{a^{n+1} - b^{n+1}}{b - a}$

  4. $\dfrac{b^{n+1} - a^{n+1}}{b - a}$


Correct Option: D
Explanation:
We know $\dfrac{1}{1-ax} = \displaystyle \sum _{r=0}^{\infty} (ax)^r$ 
$1+ax+a^2 x^2+............$
$\dfrac {1}{1-bx} = 1+bx+b^2 x^2 +..........$
So $\dfrac{1}{(1-ax)} \times \dfrac{1}{(1-bx)} = \displaystyle \sum _{r=0}^{\infty} (ax)^r \times \displaystyle \sum _{r=0}^{\infty}(bx)^r$
coefficient of $x^n$ is given by $= a^n + a^{n-1} b + .... + b^n$
given term is $g.p$ will ratio of $b/a$
So $^an = \dfrac{a^n \left(1-\left(\dfrac{b}{a}\right)^{n+1}\right)}{1-b/a} = \dfrac{a^n(a^{n+1} - b^{n+1} )/a^{n+1}}{\dfrac{(a-b)}{a}}$
$= \dfrac{a^{n+1}-b^{n+1}}{a-b}$
$= \dfrac{b^{n+1}-a^{n+1}}{b-a}$

If the sum of an infinite $G.P.$ is $1$ and the second term is $'x'$.

Then the range of $'x'$ is

  1. $\left( 0,\dfrac { 1 }{ 4 } \right]$

  2. $\left[ -2,\dfrac { 1 }{ 4 } \right]$

  3. $(-2, 0)$

  4. $[-2, 0]$


Correct Option: A

The value of $a^{\log _{2}}x$, where $a=0.2,b=\sqrt {5},x=\dfrac {1}{4}+\dfrac {1}{8}+\dfrac {1}{16}+.....$ to $\infty $ is

  1. $1$

  2. $2$

  3. $\dfrac {1}{2}$

  4. $4$


Correct Option: D
Explanation:

$x=\cfrac { 1 }{ 4 } +\cfrac { 1 }{ 8 } +\cfrac { 1 }{ 16 } +....,a=0.2,b\sqrt { 5 } \ x\cfrac { \cfrac { 1 }{ 4 }  }{ 1-\cfrac { 1 }{ 2 }  } \ x=\cfrac { 1 }{ 2 } \ { a }^{ \log _{ b }{ x }  }=(0.2)^{ \log _{ \sqrt { 5 }  }{ \cfrac { 1 }{ 2 }  }  }\ =(\cfrac { 1 }{ 2 } )^{ \log _{ \sqrt { 5 }  }{ 0.2 }  }\ =(\cfrac { 1 }{ 2 } )^{ -2 }\ a^{ \log _{ b }{ x }  }=4$

If $0<x,y,a,b<1$,then the sum of infinite terms of the series $\sqrt x (\sqrt a  + \sqrt x ) + \sqrt x (\sqrt {ab}  + \sqrt {xy} ) + \sqrt x (b\sqrt a  + y\sqrt x ) + .......$ is

  1. $\dfrac{{\sqrt {ax} }}{{1 + \sqrt b }} + \dfrac{x}{{1 + \sqrt y }}$

  2. $\dfrac{{\sqrt x }}{{1 + \sqrt b }} + \dfrac{{\sqrt x }}{{1 + \sqrt y }}$

  3. $\dfrac{{\sqrt x }}{{1 - \sqrt b }} + \dfrac{{\sqrt x }}{{1 - \sqrt y }}$

  4. $\dfrac{{\sqrt {ax} }}{{1 - \sqrt b }} + \dfrac{x}{{1 - \sqrt y }}$


Correct Option: D
Explanation:

$0<x,y,a,b<1\ S=\sqrt { x } (\sqrt { a } +\sqrt { x } )+\sqrt { x } (\sqrt { ab } +\sqrt { xy } )+\sqrt { x } (b\sqrt { a } +y\sqrt { x } )\ =\sqrt { x } \left[ (\sqrt { a } +\sqrt { ab } +b\sqrt { a } ....) \right] +(\sqrt { x } +\sqrt { xy } +y\sqrt { x } )\ =\sqrt { x } \left[ (\cfrac { \sqrt { a }  }{ 1-\sqrt { b }  } )+\cfrac { \sqrt { x }  }{ 1-\sqrt { y }  }  \right] \ S=\cfrac { \sqrt { ax }  }{ 1-\sqrt { b }  } +\cfrac { x }{ 1-\sqrt { y }  } $

If $A = 1 + {r^a} + {r^{2a}} + {r^{3a}}......\infty $ and $B = 1 + {r^b} + {r^{2b}}......\infty$ then$\dfrac{a}{b} = $

  1. $\dfrac{\log{\left({A-1}\right)}}{\log{\left({B-1}\right)}}$

  2. $\dfrac{\log{\left(\dfrac{A-1}{A}\right)}}{\log{\left(\dfrac{B-1}{B}\right)}}$

  3. $\dfrac{\log{\left({A}\right)}}{\log{\left({B}\right)}}$

  4. $\dfrac{\log{\left({B}\right)}}{\log{\left({A}\right)}}$


Correct Option: B
Explanation:

If $\left|Common\, ratio\right|<1$, then sum of infinite terms of G.P. is

$S=\dfrac{First\,  Term}{1\,−\,Common \,Ratio}$

$A=1+{r}^{a}+{r}^{2a}+{r}^{3a}+...$

Common ratio$=\dfrac{{r}^{a}}{1}={r}^{a}$

$A=\dfrac{1}{1−{r}^{a}}$

$\Rightarrow\,1−{r}^{a}=\dfrac{1}{A}$

$\Rightarrow\,{r}^{a}=1−\dfrac{1}{A}$

$\Rightarrow\,{r}^{a}=\dfrac{A−1}{A}$

Take $\log$ on both

$\log{\left({r}^{a}\right)}=\log{\left(\dfrac{A−1}{A}\right)}$

As $\log{\left({m}^{n}\right)}=n\log{\left(m\right)}$

$\Rightarrow\,a\log{\left(r\right)}=\log{\left(\dfrac{A−1}{A}\right)}$  ....$(i)$

$B=1+{r}^{b}+{r}^{2b}+{r}^{3b}+...$

Common ratio$=\dfrac{{r}^{b}}{1}={r}^{b}$

$B=\dfrac{1}{1−{r}^{b}}$

$\Rightarrow\,1−{r}^{b}=\dfrac{1}{B}$

$\Rightarrow\,{r}^{b}=1−\dfrac{1}{B}$

$\Rightarrow\,{r}^{b}=\dfrac{B−1}{B}$

Take $\log$ on both

$\log{\left({r}^{b}\right)}=\log{\left(\dfrac{B−1}{B}\right)}$

As $\log{\left({m}^{n}\right)}=n\log{\left(m\right)}$

$\Rightarrow\,b\log{\left(r\right)}=\log{\left(\dfrac{B−1}{B}\right)}$  ....$(ii)$

$\dfrac{(i)}{(ii)}=\dfrac{a\log{r}}{b\log{r}}=\dfrac{\log{\left(\dfrac{A-1}{A}\right)}}{\log{\left(\dfrac{B-1}{B}\right)}}$

$\therefore\,\dfrac{a}{b}=\dfrac{\log{\left(\dfrac{A-1}{A}\right)}}{\log{\left(\dfrac{B-1}{B}\right)}}$

The sum of the terms of an infinitely decreasing G.P. is $S$. The sum of the squares of the terms of the progression is -

  1. $\dfrac{S}{{2S - 1}}$

  2. $\dfrac{{{S^2}}}{{2S - 1}}$

  3. $\dfrac{S}{{2 - S}}$

  4. ${S^2}$


Correct Option: B
Explanation:

$1,r,r^2,r^3..........,(r<0)$


$S _{\infty}=\dfrac {a}{1-r}$

$S=\dfrac {1}{1-r}$


$r=1-\dfrac {1}{S}=\dfrac {S-1}{S}$

$S _{\infty}=1^2+r^2+r^4+r^6+..........$

$S _{\infty}=\dfrac {1}{1-r^2}$

       $=\dfrac {1}{1- \left (\dfrac {S-1}{S}\right )^2}$

       $=\dfrac {S^2}{S^2-S^2-1+2S}$

        $=\dfrac {S^2}{2S-1}$

In a GP the product of the first four terms is 4 and the second term is the reciprocal of the fourth term. The sum of the GP up to infinite terms is-

  1. $2$

  2. $\dfrac{2}{3}$

  3. $-2$

  4. 6


Correct Option: A,B,C
Explanation:

Let the four terms be $\dfrac { a }{ { r }^{ 2 } } ,\dfrac { a }{ r } ,a,a{ r }$.
Therefore, $\frac { a }{ { r }^{ 2 } } *\frac { a }{ r } *a*a{ r }=4$
$\Rightarrow \dfrac{a^{ 4 }}{r^2}=4$
Also, given that second term is a reciprocal of the fourth term.
So, $\dfrac{a}{ r }=\dfrac { 1 }{ ar } $
$a=\pm 1, r=\pm \dfrac{1}{2}$
Since the sum of infinite terms of G.P is given by $\frac { a }{ 1-r } $. So, by substituting the values of $a$ and $r$; we get
$\frac { a }{ 1-r } =\pm 2$ or $\pm \dfrac{2}{3}$

Sum to infinity of a G.P is $15$, whose first term is $a$ then a MUST satisfy the inequality given by

  1. $0< a< 130$

  2. $0< a< 30$

  3. $0< a< 15$

  4. $0< a< 100$


Correct Option: C
Explanation:

given $a+ar+ar^{2}+ar^{3}+.......=15$ 

where $0 < r < 1  \dfrac{a}{1-r}=15$ 
$(\because |r|\geqslant 1$, geometric series is divergent $)$
$a=15(1-r)$
when $0 < r < 1 $
$\Rightarrow -1 < -r < 0.$
$\Rightarrow  0 < 1-r < 1$
$\therefore  0 < 15(1-r) < 15$
$\Rightarrow 0 < a < 15$

If $x=\sqrt{4}.\sqrt[4]{4}. \sqrt[8]{4}.\sqrt[16]{4}........ \infty$, then 

  1. $x^2-8x+16=0$

  2. $x^2-3x+2=0$

  3. $x^2-5x+4=0$

  4. $x^2+5x+4=0$


Correct Option: A
Explanation:

$\begin{array}{l} x={ 4^{ 1/2 } }{ 4^{ 1/4 } }{ 4^{ 1/8 } }\cdots \infty  \ ={ 4^{ \frac { 1 }{ 2 } +\frac { 1 }{ 4 } +\cdots +\infty  } } \ ={ 4^{ \frac { { 1/2 } }{ { 1-\frac { 1 }{ 2 }  } }  } } \ =4 \ { \left( { x-4 } \right) ^{ 2 } }=0 \ { x^{ 2 } }+16-8x=0 \end{array}$

The sum of  $3,1,\dfrac 13 ,....$ is

  1. $\dfrac 52$

  2. $\dfrac 92$

  3. $\dfrac 72$

  4. $\dfrac {11}2$


Correct Option: B
Explanation:

Given series is $3,1,\dfrac 13,..$


Given series is in GP.

The common ratio is given as $\dfrac{1}{3}$

The sum of infinite terms is  $\dfrac{a}{1-r}$

$\implies  \dfrac 3{1-\dfrac 13}$

$\implies \dfrac{3}{\dfrac 23}=\dfrac 92$


If the sum of an infinite GP is 20 and sum of their square is 100 then common ratio will be 

  1. $\dfrac{1}{2}$

  2. $\dfrac{1}{4}$

  3. $\dfrac{3}{5}$

  4. $1$


Correct Option: C
Explanation:
Let $a,ar,a{r}^{2},...$ to $\infty$
Now, sum of infinite G.P$=20$
$\Rightarrow \dfrac{a}{1-r}=20$          .........$\left(1\right)$
where each term of the above G.P is squared, then the progression becomes
${a}^{2},{a}^{2}{r}^{2},{a}^{2}{r}^{4},.....$
Now, first term $A={a}^{2}$ and common ratio$R={r}^{2}$
Sum of above G.P$=100$
$\Rightarrow \dfrac{A}{1-R}=100$
$\Rightarrow \dfrac{{a}^{2}}{1-{r}^{2}}=100$     ....$\left(2\right)$
Squaring $\left(1\right)$ we get
$\dfrac{{a}^{2}}{{\left(1-r\right)}^{2}}=400$    ....$\left(3\right)$
Dividing eqn$\left(2\right)$ by $\left(3\right)$ we get
$\dfrac{{\left(1-r\right)}^{2}}{1-{r}^{2}}=\dfrac{400}{100}=4$
$\Rightarrow \dfrac{{\left(1-r\right)}^{2}}{\left(1-r\right)\left(1+r\right)}=4$
$\Rightarrow \dfrac{1-r}{1+r}=4$
$\Rightarrow 4-4r=1+r$
$\Rightarrow 5r=3$
$\therefore r=\dfrac{3}{5}$

For first $n$ natural numbers we have the following results with usual notations $ \displaystyle \sum _{r=1}^{n}r =\frac{n(n+1)}{2}, \sum _{r=1}^{n}r^{2} =\frac{n(n+1)(2n+1)}{6},\sum _{r=1}^{n}r^{3}=\left ( \sum _{r=1}^{n}r \right )^{2}$ If $\displaystyle a _{1}a _{2}....a _{n} \in A.P $ then sum to $n$ terms of the sequence $\displaystyle \frac{1}{a _{1}a _{2}},\frac{1}{a _{2}a _{3}},...\frac{1}{a _{n-1}a _{n}}$ is equal to $\displaystyle \frac{n-1}{a _{1}a _{n}}$
 and the sum to $ n$ terms of a $G.P$ with first term '$a$' & common ratio '$r$' is given by  $\displaystyle S _{n}= \frac{lr-a}{r-1}$ for $ r \neq 1 $ for $ r =1 $ sum to $n$ terms of same $G.P.$ is $n$ $a$, where the sum to infinite terms of$G.P.$ is the limiting value of
 $\displaystyle \frac{lr-a}{r-1} $ when $\displaystyle n \rightarrow \infty ,\left |  r \right | < l $ where $l$ is the last term of $G.P.$  On the basis of above data answer the following questionsThe sum to infinite terms of the series $\displaystyle \frac{1}{2}+\frac{1}{6}+\frac{1}{18}+.. $ is equal to ?

  1. $\displaystyle \frac{4}{3}$

  2. $\displaystyle \frac{3}{4}$

  3. $\displaystyle \frac{8}{3}$

  4. Does not exit


Correct Option: B
Explanation:

Let, ${ S } _{ \infty  }=\dfrac { 1 }{ 2 } +\dfrac { 1 }{ 6 } +d\frac { 1 }{ 18 } +..\infty $

$\Rightarrow { S } _{ \infty  }=\dfrac { 1 }{ 2 } \left( 1+\dfrac { 1 }{ 3 } +\dfrac { 1 }{ { 3 }^{ 2 } } +....\infty  \right) $

As we know that, sum of infinite G.P series $=\dfrac { a }{ 1-r } $

Therefore, $ { S } _{ \infty  }=\dfrac { 1 }{ 2 } \left( \dfrac { 1 }{ 1-\left( 1/3 \right)  }  \right) =\dfrac { 3 }{ 4 } $

Ans: B

If $\displaystyle x=\sum _{a=0}^{\infty }a^{n},y=\sum _{a=0}^{\infty }b^{n},z=\sum _{a=0}^{\infty }c^{n}$ Where $a,b,c $ are in A.P and $\displaystyle \left | a \right |<1,\left | b \right |<1,\left | c \right |<1$ then $x,y,z$ are in

  1. H.P

  2. Arithmetic-Geometric progression

  3. A.P

  4. G.P


Correct Option: A
Explanation:

Given $\displaystyle \left | a \right |< 1,\left | b \right |< 1,\left | c \right |< 1,\ \ a,b,c \in A.P$


and $\displaystyle \sum _{n=0}^{\infty }a^{n}=\frac{1}{1-a},\sum _{n=0}^{\infty }b^{n}=\frac{1}{1-b},\sum _{r=0}^{\infty }c^{n}=\frac{1}{1-c}$

$\displaystyle \therefore x=\frac{1}{1-a},y=\frac{1}{1-b},c=\frac{1}{1-c}$

$\displaystyle \Rightarrow a=\frac{x-1}{x},b=\frac{y-1}{y},c=\frac{z-1}{z}$

$\displaystyle \because 2b=a+c \ as \ a,b,c \in A.P$

$\displaystyle 2\left ( \frac{y-1}{y} \right )=\frac{x-1}{x}+\frac{z-1}{z}\Rightarrow \frac{2}{y}=\frac{1}{x}+\frac{1}{z}$

$\displaystyle \Rightarrow x,y,z \in H.P$

If $R \subset\left ( 0,\pi  \right )$ denote the set of values of which satisfies the equation $ \displaystyle 2^{\left ( 1+\left | \cos x \right |+\left | cos^{2}x \right |+\left | cos^{3}x \right | \right )+\left | cos^{4}x  \right |...............\infty}=4$ then $R$ equals

  1. $\displaystyle\left { -\frac{\pi }{3} \right }$

  2. $\displaystyle\left { \frac{\pi }{3},\frac{2\pi }{3} \right }$

  3. $\displaystyle\left { \frac{-\pi }{3},\frac{2\pi }{3} \right }$

  4. $\displaystyle\left { \frac{\pi }{3},\frac{-2\pi }{3} \right }$


Correct Option: B
Explanation:

${ 2 }^{ \left( 1+\left| \cos { x }  \right| +\left| \cos ^{ 2 }{ x }  \right| +.........\infty  \right)  }={ 2 }^{ 2 }\ \Rightarrow 1+\left| \cos { x }  \right| +\left| \cos ^{ 2 }{ x }  \right| +.........\infty =2\ \Rightarrow \dfrac { 1 }{ 1-\left| \cos { x }  \right|  } =2\ \Rightarrow 1-\left| \cos { x }  \right| =\dfrac { 1 }{ 2 } \ \Rightarrow \left| \cos { x }  \right| =\dfrac { 1 }{ 2 } \ \Rightarrow x=\dfrac { \pi  }{ 3 } ,\dfrac { 2\pi  }{ 3 } $
  in the range $\left( 0,\pi  \right) $

The sum of the series
$\dfrac { 1 } { 1.2 } - \dfrac { 1 } { 2.3 } + \dfrac { 1 } { 3.4 } \ldots \ldots \ldots$  up to  $\infty$  is equal to

  1. $\log _{ { { e } } } \left( \dfrac { 4 }{ { e } } \right) $

  2. $2 \log _ { e } 2$

  3. $\log _ { e } 2 - 1$

  4. $\log _ { e } 2$


Correct Option: A

The sum of the infinite series, ${ 1 }^{ 2 }-\frac { { 2 }^{ 2 } }{ 5 } +\frac { { 3 }^{ 2 } }{ { 5 }^{ 2 } } -\frac { { 4 }^{ 2 } }{ { 5 }^{ 3 } } +\frac { { 5 }^{ 2 } }{ { 5 }^{ 4 } } -\frac { { 6 }^{ 2 } }{ { 5 }^{ 5 } } +.........$ is :

  1. $\frac { 1 }{ 2 } $

  2. $\frac { 25 }{ 24 } $

  3. $\frac { 25 }{ 54 } $

  4. $\frac { 125 }{ 252 } $


Correct Option: A

The first term of an infinitely decreasing G.P. is unity and its sum is S. The sum of the squares of the terms of the progression is

  1. $\displaystyle \frac {S}{2S-1}$

  2. $\displaystyle \frac {S^2}{2S-1}$

  3. $\displaystyle \frac {S}{2-S}$

  4. $S^2$


Correct Option: B
Explanation:

Let common ratio is $r<1$
Then G.P is $1,r,{ r }^{ 2 },{ r }^{ 3 },...\infty $
$S=1+r+{ r }^{ 2 }+{ r }^{ 3 }+...\infty $
$\displaystyle \Rightarrow S=\frac { 1 }{ 1-r } $
Then G.P formed by squaring the terms 
$1,{ r }^{ 2 },{ r }^{ 4 },{ r }^{ 6 },...\infty $
$\displaystyle { S }'=\frac { 1 }{ 1-{ r }^{ 2 } } =\frac { 1 }{ \left( 1-r \right) \left( 1+r \right)  } =\frac { { S }^{ 2 } }{ 2S-1. } $

 If  $0<\phi < \pi /2,$   and
 $x= \sum _{n=0}^{\infty} \cos ^{2n} \phi$, $ y=\sum _{n=0}^{\infty } \sin ^{2n} \phi$                     
and $z=\sum _{n=0}^{\infty} \cos ^{2n} \phi \sin ^{2n} \phi $ 
then

  1. xyz $=$xz+y

  2. xyz$=$xy+z

  3. xyz$=$x+y+z

  4. xy$=$yz+z


Correct Option: B,C
Explanation:

The series x converges to a value $\dfrac{1}{1-{\cos^2 \phi}}\;=\;{cosec^2 \phi}$

Similarly y converges to  a value ${sec^2 \phi} $

In a similar fashion.. z converges to $\dfrac{{cosec^2 \phi}{sec^2 \phi}}{{cosec^2 \phi}{sec^2 \phi} - 1}$
 $ z = \dfrac{xy}{xy - 1} $

$xy + z = xyz&gt;$; Option B.
And, also
$x + y = xy$
$\therefore xyz = x+ y+ z&gt;$; Option C.

Find the sum of the infinite geometric series where the beginning term is $-1$ and the common ratio is $\dfrac{1}{2}$.

  1. $1$

  2. $-1$

  3. $2$

  4. $-2$


Correct Option: D
Explanation:

Given that first term $a=-1$ and common ratio is $r=\dfrac{1}{2}$

We know $\text{sum} = \dfrac{a}{1-r}$
$\Rightarrow \text{sum} = \dfrac{-1}{1-\frac{1}{2}}$
$\Rightarrow \text{sum} = \dfrac{-1}{\frac{1}{2}}$
$\Rightarrow \text{sum} = -2$

$1 + x + x^2 + x^3 +......$ = ?

  1. $\dfrac{1}{1-x}$

  2. $\dfrac{1}{1-x^2}$

  3. $\dfrac{1}{1-x^3}$

  4. $\dfrac{x}{1-x}$


Correct Option: A
Explanation:

Sum of a GP $a,ar,ar^2......$

$=\dfrac{a(r^n -1)}{r-1}$    (for $n$ terms)
For the given series,
$a=1, r=x, n \to \infty$
Sum of the series is:
$\text{sum} = \lim _{n \to \infty} \dfrac{x^n-1}{x-1}$
For $x>1$ 
$\text{sum} \to \infty$
For $x<1$
$x^n \to 0$
$\Rightarrow \text{sum} = \dfrac{-1}{x-1}$
$\Rightarrow \text{sum} = \dfrac{1}{1-x}$

If $a=\sum _{ n=0 } ^{\infty  }{x^n } ,b=\sum _{n=0  }^{ \infty  }{ y^n } , c=\sum _{n=0  }^{ \infty  }{ (xy)^n } $ where $|x| ,| y| < 1$ ; then

  1. $abc = a + b + c$

  2. $ab + bc = ac + b$

  3. $ac + bc = ab + c$

  4. $ab + ac = bc + a$


Correct Option: C
Explanation:

Clearly every summation is infinite series,
$a=\cfrac{1}{1-x}, b = \cfrac{1}{1-y}$ and $c=\cfrac{1}{1-xy}$
or $x=1-\cfrac{1}{a}, y=1-\cfrac{1}{b}$
Simplifying above equation we get, $ac+bc=ab+c$

Sum to infinity of the series $\displaystyle \frac { 2 }{ 3 } -\frac { 5 }{ 6 } +\frac { 2 }{ 3 } -\frac { 11 }{ 24 } +...$ is

  1. $\displaystyle \frac { 4 }{ 9 } $

  2. $\displaystyle \frac { 1 }{ 3 } $

  3. $\displaystyle \frac { 2 }{ 9 } $

  4. none of these


Correct Option: C
Explanation:

Let $\displaystyle S=\frac { 2 }{ 3 } -\frac { 5 }{ 6 } +\frac { 2 }{ 3 } -\frac { 11 }{ 24 } +...$ to $\infty$   ...(1)


Multiplying both sides by $\displaystyle -\frac { 1 }{ 2 } $, the common ratio $G.P.$


$\displaystyle -\frac { 1 }{ 2 } S=-\frac { 2 }{ 6 } +\frac { 5 }{ 12 } -\frac { 8 }{ 24 } +...$ to $\infty$    ....(2)

Subtracting (2) from (1), we have

$\displaystyle \frac { 3 }{ 2 } S=\frac { 2 }{ 3 } -\frac { 3 }{ 6 } +\frac { 3 }{ 12 } -\frac { 3 }{ 24 } +...$ to $\infty$

$\displaystyle =\frac { 2 }{ 3 } -\left( \frac { 1 }{ 2 } -\frac { 1 }{ 4 } +\frac { 1 }{ 8 } +... \right) $

$\displaystyle =\dfrac { 2 }{ 3 } -\dfrac { \dfrac { 1 }{ 2 }  }{ 1-\left( -\dfrac { 1 }{ 2 }  \right)  } =\dfrac { 2 }{ 3 } -\dfrac { 1 }{ 3 } =\dfrac { 1 }{ 3 } $

$\displaystyle \therefore S=\frac { 1 }{ 3 } \times \frac { 2 }{ 3 } =\frac { 2 }{ 9 } $

If $S$ is the sum to infinity of a GP, whose first term is $a$, then the sum of the first $ n$  terms is

  1. $\displaystyle S\left ( 1-\frac{a}{S} \right )^{n}$

  2. $\displaystyle S\left [ 1-\left ( 1-\frac{a}{S} \right )^{n} \right ]$

  3. $\displaystyle a\left [ 1-\left ( 1-\frac{a}{S} \right )^{n} \right ]$

  4. none of these


Correct Option: B
Explanation:

Let r be the common ration.
GIven, $S _\infty=S=\frac{a}{1-r}$
$\Rightarrow 1-r=\frac aS$
$\Rightarrow r=1-\frac aS$
Now, sum of n terms is given by
$S _n=\dfrac{a(1-r^n)}{1-r}$


       $=\dfrac{a(1-(1-\frac aS)^n)}{1-(1-\frac aS)}$

       $=\dfrac{a(1-(1-\frac aS)^n)}{\frac aS}$


       $=S[1-(1-\frac aS)^n]$
Option B is correct.

$\displaystyle2+1+\frac{1}{2}+\frac{1}{4}+\cdots\cdots\infty$ is

  1. 1

  2. 2

  3. 3

  4. 4


Correct Option: D
Explanation:

The given series is a Geometric Progression, with first terms $ a =2$ and common ratio $ r = \dfrac {T _2}{T _1} = \dfrac {1}{2} $

For a GP, sum to infinity is given by the formula $ \dfrac {a}{1-r} $

So, for the given series, $ S _\infty  = \dfrac {2}{1-\dfrac {1}{2}} = 4 $

What is the sum of the infinite geometric series where the beginning term is $2$ and the common ratio is $3$?

  1. $1$

  2. $-1$

  3. $2$

  4. $-2$


Correct Option: B
Explanation:

From the given information, we have
first term $=a=2 $, common ratio $r=3$
We know $S = \dfrac{a}{1-r}$
Therefore, $S = \dfrac{2}{1-3}$
$\Rightarrow S = -1$

The value of the infinite product $6^{\frac{1}{2}}\times 6^{\frac{1}{2}}\times 6^{\frac{3}{8}}\times 6^{\frac{1}{4}}\times .........$ is

  1. 6

  2. 36

  3. 216

  4. $\infty$


Correct Option: B
Explanation:
$6^{\frac12}\times6^{\frac12}\times6^{\frac38}\times6^{\frac14}\times...............$
$=6^{\frac12+\frac24+\frac38+\frac4{16}+...................}$
$=6^{\frac12\left(1+\frac22+\frac3{2^2}+\frac4{2^3}+......................\right)}$

Let $S=1+\cfrac22+\cfrac3{2^2}+\cfrac4{2^3}+............$, then
$\cfrac S2=S-\cfrac S2$
or, $\cfrac S2=1+\cfrac12+\cfrac1{2^2}+\cfrac1{2^3}+...........$
or, $\cfrac S2=\cfrac1{1-\cfrac12}$ ..... [Using sum of infinite terms of an G.P]
or, $S=4$
Then we have,
$6^{\frac12}\times6^{\frac12}\times6^{\frac38}\times6^{\frac14}\times...............$
$=6^{\frac12\times4}=6^2=36$
Hence, B is the correct option.

Calculate the sum of the infinite series: $1 - \dfrac {1}{3} + \dfrac {1}{9} - \dfrac {1}{27} + .....$.

  1. $\dfrac {2}{3}$

  2. $\dfrac {3}{4}$

  3. $1$

  4. $\dfrac {4}{3}$

  5. $\dfrac {3}{2}$


Correct Option: B
Explanation:

Given series is $1,-\dfrac{1}{3},+\dfrac{1}{9},-\dfrac{1}{27}......................$

Then common ratio $=$ $ (-\dfrac{1}{3})/1=-\dfrac{1}{3}$
Then sum of infinite series $S=$ $\dfrac{a _{1}}{1-r}=\dfrac{1}{1-(-\frac{1}{3})}=\dfrac{1}{\frac{1+3}{3}}=\dfrac{3}{4}$

Calculate the sum of the infinite geometric series $2+\left(-\displaystyle\frac{1}{2}\right)+\left(\displaystyle\frac{1}{8}\right)+\left(-\displaystyle\frac{1}{32}\right)+...$

  1. $1\displaystyle\frac{3}{8}$

  2. $1\displaystyle\frac{2}{5}$

  3. $1\displaystyle\frac{1}{2}$

  4. $1\displaystyle\frac{3}{5}$

  5. $1\displaystyle\frac{5}{8}$


Correct Option: D
Explanation:

Given the geometric series is $2,\left ( -\dfrac{1}{2} \right ),\left ( \dfrac{1}{8} \right ),\left ( -\dfrac{1}{32} \right ).......................$

Then common ratio $=-\dfrac{1}{4}$
And first term is $2$.
Then sum of the infinite geometric series $=$ $S=\dfrac{a _{1}}{1-r}=\dfrac{2}{1-(-\frac{1}{4})}=\dfrac{2\times 4}{4+1}=\dfrac{8}{5}=1\dfrac{3}{8}$

The sum of first $n$ terms of an infinite G.P. is

  1. $S = \dfrac{a}{1-r}$

  2. $S _n = \dfrac{a _1(1-r^n)}{1-r}$

  3. $S = \dfrac{an}{1-r}$

  4. $S _n = \dfrac{a _1(1-r^n)}{1+r}$


Correct Option: A
Explanation:

Sum of GP $=\dfrac{a(r^n -1)}{r-1}$

$a= $ first term
$r=$ common ratio
For $n \to \infty$
$r^n = 0$ for $r<1$
$r^n \to \infty $ for $r>1$
Thus $\text{sum} =\dfrac {a}{r-1}$

If ${S} _{p}$ denote the sum of the series $1+{r}^{p}+{r}^{2p}+..$ upto infinity and ${X} _{p}$ be the sum of the series $1-{r}^{p}+{r}^{2p}-..$ upto infinity then $\left( r\in \left( -1,1 \right) -\left{ 0 \right}  \right)$

  1. ${S} _{p}+{X} _{p}={2X} _{2p}$

  2. ${S} _{p}+{X} _{p}={2S} _{2p}$

  3. ${S} _{p}+{X} _{p}={S} _{2p}$

  4. $None\ of\ these$


Correct Option: A
Explanation:

$\begin{array}{l} { S _{ p } }\, \, is\, \, m-1+{ r^{ p } }+{ r^{ 2p } }... \ { X _{ p } }=1-{ r^{ p } }+{ r^{ 2p } } \ { S _{ p } }=\frac { 9 }{ { 1-R } } =\frac { 1 }{ { 1-{ r^{ p } } } }  \ { X _{ p } }=\frac { 1 }{ { 1+{ r^{ p } } } }  \ { S _{ p } }+{ X _{ p } }=\frac { 1 }{ { 1-{ r^{ p } } } } +\frac { 1 }{ { 1+{ r^{ p } } } }  \ =\frac { { 1+{ r^{ p } }+1 } }{ { \left( { 1-{ r^{ p } } } \right) \left( { 1+{ r^{ p } } } \right)  } } =\frac { 2 }{ { 1-{ r^{ 2p } } } }  \ =2{ S _{ p } } \  \end{array}$

The sum of an infinite geometric series whose first term is a and common ratio is r is given by

  1. $\displaystyle S _{\infty} = \frac{1}{a - r}$

  2. $\displaystyle S _{\infty} = \frac{1}{r-a}$

  3. $\displaystyle S _{\infty} = \frac{a}{1 - r}$

  4. $\displaystyle S _{\infty} = \frac{1-r}{a}$


Correct Option: C
Explanation:

Lets say the sum of infinite geometric series is:

$S _\infty=a + ar + ar^{2} + ar^{3}...... ar^{n}.....\infty$
Let us multiply r on both sides

$rS _\infty=ar+ar^{2}+.................\infty$
Let us subtract $rS _\infty$ to from $S _\infty$
So we can write

$S _\infty-rS _\infty=a$
$S _\infty(1-r)=a$

$S _\infty=\dfrac{a}{1-r}$

If $S _{1}, S _{2}, S _{3}$ are respectively the sum of n, 2n and 3n terms of a G.P. Then  $S _{1}(S _{3}-S _{2}) = (S _{2} -S _{1})^{2}$.

  1. True

  2. False


Correct Option: A
Explanation:

${ S } _{ 1 }=$Sum of n terms in G.P.

${ S } _{ 2 }=$Sum of 2n terms in G.P.
${ S } _{ 3 }=$4Sum of 3n terms in G.P.
${ S } _{ 1 }=\cfrac { a({ r }^{ n }-1) }{ r-1 } ,{ S } _{ 2 }=\cfrac { a({ r }^{ 2n }-1) }{ r-1 } ,{ S } _{ 3 }=\cfrac { a({ r }^{ 3n }-1) }{ r-1 } $
a is a first term and r is a common ratio.
${ S } _{ 3 }-{ S } _{ 2 }=\cfrac { a({ r }^{ 3n }-{ r }^{ 2n }) }{ r-1 } \ { S } _{ 1 }({ S } _{ 3 }-{ S } _{ 2 })=\cfrac { { a }^{ 2 }({ r }^{ 4n }-2{ r }^{ 3n }+{ r }^{ 2n }) }{ { (r-1 })^{ 2 } } \ =\cfrac { { a }^{ 2 }({ r }^{ 2n }-{ r }^{ n })^{ 2 } }{ { (r-1 })^{ 2 } } \ =\cfrac { { a }^{ 2 }(({ r }^{ 2n }-{ 1)(r }^{ n }-1))^{ 2 } }{ { (r-1 })^{ 2 } } \ =\left[ \cfrac { { a }({ r }^{ 2n }-{ 1)-a(r }^{ n }-1) }{ { (r-1 }) }  \right] ^{ 2 }\ { S } _{ 1 }({ S } _{ 3 }-{ S } _{ 2 })=({ S } _{ 2 }-{ S } _{ 1 })^{ 2 }$

If $|x| > 1$, then
$\left(1-\dfrac{1}{x}\right)+\left(1-\dfrac{1}{x}\right)^2+\left(1-\dfrac{1}{x}\right)^3+.....=$

  1. $x-1$

  2. $x+1$

  3. $x$

  4. $\dfrac{1}{x-1}$


Correct Option: A
Explanation:

$S=\left( 1-\cfrac { 1 }{ x }  \right) +{ \left( 1-\cfrac { 1 }{ x }  \right)  }^{ 2 }+{ \left( 1-\cfrac { 1 }{ x }  \right)  }^{ 3 }+........$
This is a G.P whose first term, $a=1-\cfrac { 1 }{ x } $ and common ration,  $r=1-\cfrac { 1 }{ x } $
${ S } _{ \infty  }=\cfrac { a }{ 1-r } $
$=\cfrac { 1-\cfrac { 1 }{ x }  }{ 1-\left( 1-\cfrac { 1 }{ x }  \right)  } $
$=\cfrac { { \left( x-1 \right)  }/{ x } }{ { \left[ x-\left( x-1 \right)  \right]  }/{ x } } $
$=\cfrac { x-1 }{ x-x+1 } $
$=x-1$

If $e^{\displaystyle \left [ \left ( \sin^{2}x + \sin^{4}x + \sin^{6}x + .... + \infty \right ) \log _{e}2\right ]}$ satisfies the equation $\displaystyle x^{2} -9x + 8 = 0$,then the value of $\displaystyle g \left ( x \right ) = \frac{\cos x}{\cos x + \sin x}$ is

  1. $\displaystyle \frac{\sqrt{3} + 1}{2}$

  2. $\displaystyle \frac{\sqrt{3} - 1}{2}$

  3. $\displaystyle 8$

  4. None of these


Correct Option: B
Explanation:

Consider, $y=exp\left[ \left( \sin ^{ 2 } x+\sin ^{ 4 } x+\sin ^{ 6 } x+....+\infty  \right) \log _{ e } 2 \right] $
$\displaystyle\Rightarrow y=exp\left[ \left( \frac { \sin ^{ 2 }{ x }  }{ 1-\sin ^{ 2 }{ x }  }  \right) \log _{ e } 2 \right] =exp\left[ \tan ^{ 2 }{ x } \log _{ e } 2 \right] ={ 2 }^{ \tan ^{ 2 }{ x }  }$
Since, $y$ satisfies $x^{ 2 }-9x+8=0$, then
        $y=1,8$
$\Rightarrow { 2 }^{ \tan ^{ 2 }{ x }  }={ 2 }^{ 0 },{ 2 }^{ 3 }$
$\Rightarrow \tan ^{ 2 }{ x } =0,3$
$\Rightarrow \tan { x } =0,\pm \sqrt { 3 } $

Now, $\displaystyle g\left( x \right) =\frac { \cos  x }{ \cos  x+\sin  x } =\frac { 1 }{ 1+\tan { x }  } =1,\frac { 1 }{ 1\pm \sqrt { 3 }  } =1,\frac { -\sqrt { 3 } -1 }{ 2 } ,\frac { \sqrt { 3 } -1 }{ 2 } $

Ans: B

lf $e^{(\cos^{2}x+\cos^{4}x+\cos^{6}x+\ldots.)\log 3}$ satisfies $y^{ 2 }-10y+9=0$ and $0\le x\le \cfrac { \pi  }{ 2 } $, then $\cot^{2}x=$

  1. $0$

  2. $1$

  3. $\dfrac12$

  4. $9$


Correct Option: A
Explanation:

$\displaystyle e^{(\cos^{2}x+\cos^{4}x+\cdots )\log{3}}=e^{\left(\dfrac{\cos^{2}x}{1-\cos^{2}x}\right)\log3.}$
$\because (\cos^{2}x+\cos^{4}x+\cdots)$ is forming an infinite G.P. $=e^{\cot^{2}x\log3.}$
$y^{2}-10y+9=0  \Rightarrow   y=9,1$
$e^{\cot^{2}x\log{3}}=9,1$
$3^{\cot^{2}x}=9,1&gt;&gt;[\because e ^{\log x}=x]$
$\Rightarrow \cot^{2}x=2,0$ but as $x\in \left[0,\dfrac{\pi }{2}\right]$
$\Rightarrow \cot^{2}x=0$

Hence, option 'A' is correct.

If the sum of an infinite $GP$ is $20$ and sum of their square is $100$ then common ration will be=

  1. $1/2$

  2. $1/4$

  3. $3/5$

  4. $1$


Correct Option: C
Explanation:
Let $G.P$ is $a, ar, ar^2,....\infty$
sum $=\dfrac {a}{1-r}=20---(1)$
If terms are required the $G.P.$
becomes : $a^2, a^2r^2, a^2 r^4,.....\infty$
Sum $=\dfrac {a^2}{1-r^2}=100---(2)$
Name dividing square of equation $(1)$ by equation $(2)$
$\dfrac {\dfrac {a^2}{1-r^2}}{\dfrac {a^2}{(1-r)^2}}=\dfrac {100}{400}$
$\Rightarrow \ \dfrac {(1-r)}{(1-r)(1+r)}=\dfrac {1}{4}$
$\Rightarrow \ \dfrac {1-r}{1+r}=\dfrac {1}{4}$
$\Rightarrow \ 4-4r=1+r$
$\Rightarrow \ 4-1=4r+r$
$\Rightarrow \ 5r=3$
$\Rightarrow \ r=\dfrac {3}{5}$

For $0 < \phi < \pi/2$ if $x=\sum _{n=0}^{\infty }\cos ^{2n} \phi, y=\sum _{n=0}^{\infty }\sin ^{2n} \phi, z=\sum _{n=0}^{\infty }\cos ^{2n} \phi \sin^{2n}\phi$, then 

  1. $xyz=xz+y$

  2. $xyz=xy+z$

  3. $xyz=x+y+z$

  4. $xyz=yz+x$


Correct Option: B
Explanation:

Given $\displaystyle x=\sum _{ n=0 }^{ \infty  }{ { \cos }^{ 2n }\phi ,\quad y=\sum _{ n=0 }^{ \infty  }{ { \sin }^{ 2n }\phi  }  } $ and $\displaystyle z=\sum _{ n=0 }^{ \infty  }{ { \cos }^{ 2n }\phi  } { \sin }^{ 2n }\phi $


\since $\displaystyle 0<\phi <\frac { \pi  }{ 2 } $, so each series is geometric series with common ratio $\displaystyle r<1$. Therefore, the series are convergent.


Now, $\displaystyle x=\frac { 1 }{ 1-{ \cos }^{ 2 }\phi  } $


$\displaystyle =\frac { 1 }{ { \sin }^{ 2 }\phi  } $      $(\because S _{ \infty  }=\dfrac { a }{ 1-r } )$


$\displaystyle y=\frac { 1 }{ 1-{ \sin }^{ 2 }\phi  } $      $(\because S _{ \infty  }=\dfrac { a }{ 1-r } )$


$\displaystyle =\frac { 1 }{ { \cos }^{ 2 }\phi  } $


$\displaystyle z=\frac { 1 }{ 1-{ \sin }^{ 2 }{ \phi \cos }^{ 2 }\phi  } $ 

   

$(\because S _{ \infty  }=\dfrac { a }{ 1-r } )$


Consider, $\displaystyle xyz=\frac { 1 }{ { \sin }^{ 2 }\phi { \cos }^{ 2 }\phi (1-{ \sin }^{ 2 }{ \phi \cos }^{ 2 }\phi ) } $      $(1)$


Also, $\displaystyle =\frac { 1 }{ { \sin }^{ 2 }\phi { \cos }^{ 2 }\phi  } +\frac { 1 }{ 1-{ \sin }^{ 2 }{ \phi \cos }^{ 2 }\phi  } $


$\displaystyle xy+z=\frac { 1-{ \sin }^{ 2 }{ \phi \cos }^{ 2 }\phi +{ \sin }^{ 2 }\phi { \cos }^{ 2 }\phi  }{ { \sin }^{ 2 }\phi { \cos }^{ 2 }\phi (1-{ \sin }^{ 2 }{ \phi \cos }^{ 2 }\phi ) } $


$\displaystyle =\frac { 1 }{ { \sin }^{ 2 }\phi { \cos }^{ 2 }\phi (1-{ \sin }^{ 2 }{ \phi \cos }^{ 2 }\phi ) } $

 

$\displaystyle =xyz$      $[From (1)]$

 

The sum of the intercepts cut off by the axes on the lines  $ x+y=a,x+y=ar,x+y=ar^{2}\ldots\ldots\ldots$ where $a\neq 0$ and $r=\displaystyle \dfrac{1}{2}$  is 

  1. $2a$

  2. $a\sqrt{2}$

  3. $2\sqrt{2}a$

  4. $ \displaystyle \dfrac{a}{\sqrt{2}}$


Correct Option: C
Explanation:

$x+y=a$

intercept cut off by the axes $=\sqrt{a^2+a^2}=\sqrt{2}a$

$\therefore$ sum of all intercepts cut off by the axes on the lines,

$x+y=a, x+y=ar,.... x+y=a^{r^n-1}$

$\sqrt{2}a, \sqrt{2}ar,.... \sqrt{2}ar^{n-1}$

$Sum=\sqrt{2}a+\sqrt{2}ar+....+\sqrt{2}a^{r^n-1}....$as

$=\sqrt{2}(\dfrac{a}{1-r})$

$=\dfrac{\sqrt{2}a}{1-r}$

Given  $\Rightarrow r=\dfrac{1}{2}$

$\therefore Sum=2\sqrt{2}a$

- Hide questions